Find the volume of the solid bounded by the xy plane, the cylinder $x^{2} + y^{2}=4$, and the plane $z+y=4$.












0












$begingroup$


Find the volume of the solid bounded by the xy plane, the cylinder $x^{2} + y^{2}=4$, and the plane $z+y=4$.





If we draw the graph, then the integral will be calculated should be



$$ int_{0}^{2} int_{0}^{sqrt{4-x^{2}}} (4-y) : dy dx $$



This would give the volume of the solid in 1st quadrant which can also be obtained through



$$ int_{0}^{2} int_{0}^{sqrt{4-y^{2}}} (4-y) : dx dy $$



which would be equal to $4pi - frac83$. Both the above equations give the same result.



But if we try to find the volume of the entire solid formed by the three curves, then the results from the above equations(after changing the limits appropriately) don't match.
$$ int_{-2}^{2} 2int_{0}^{sqrt{4-x^{2}}} (4-y) : dy dx = 2(8pi-frac{16}{3})$$
$$ int_{-2}^{2} 2int_{0}^{sqrt{4-y^{2}}} (4-y) : dx dy = 16pi$$



why the results of the these two equations don't match? Is there some problem with the limits I've set or something else? Please help



PS: There is a similar question, but my query is different.










share|cite|improve this question











$endgroup$

















    0












    $begingroup$


    Find the volume of the solid bounded by the xy plane, the cylinder $x^{2} + y^{2}=4$, and the plane $z+y=4$.





    If we draw the graph, then the integral will be calculated should be



    $$ int_{0}^{2} int_{0}^{sqrt{4-x^{2}}} (4-y) : dy dx $$



    This would give the volume of the solid in 1st quadrant which can also be obtained through



    $$ int_{0}^{2} int_{0}^{sqrt{4-y^{2}}} (4-y) : dx dy $$



    which would be equal to $4pi - frac83$. Both the above equations give the same result.



    But if we try to find the volume of the entire solid formed by the three curves, then the results from the above equations(after changing the limits appropriately) don't match.
    $$ int_{-2}^{2} 2int_{0}^{sqrt{4-x^{2}}} (4-y) : dy dx = 2(8pi-frac{16}{3})$$
    $$ int_{-2}^{2} 2int_{0}^{sqrt{4-y^{2}}} (4-y) : dx dy = 16pi$$



    why the results of the these two equations don't match? Is there some problem with the limits I've set or something else? Please help



    PS: There is a similar question, but my query is different.










    share|cite|improve this question











    $endgroup$















      0












      0








      0


      1



      $begingroup$


      Find the volume of the solid bounded by the xy plane, the cylinder $x^{2} + y^{2}=4$, and the plane $z+y=4$.





      If we draw the graph, then the integral will be calculated should be



      $$ int_{0}^{2} int_{0}^{sqrt{4-x^{2}}} (4-y) : dy dx $$



      This would give the volume of the solid in 1st quadrant which can also be obtained through



      $$ int_{0}^{2} int_{0}^{sqrt{4-y^{2}}} (4-y) : dx dy $$



      which would be equal to $4pi - frac83$. Both the above equations give the same result.



      But if we try to find the volume of the entire solid formed by the three curves, then the results from the above equations(after changing the limits appropriately) don't match.
      $$ int_{-2}^{2} 2int_{0}^{sqrt{4-x^{2}}} (4-y) : dy dx = 2(8pi-frac{16}{3})$$
      $$ int_{-2}^{2} 2int_{0}^{sqrt{4-y^{2}}} (4-y) : dx dy = 16pi$$



      why the results of the these two equations don't match? Is there some problem with the limits I've set or something else? Please help



      PS: There is a similar question, but my query is different.










      share|cite|improve this question











      $endgroup$




      Find the volume of the solid bounded by the xy plane, the cylinder $x^{2} + y^{2}=4$, and the plane $z+y=4$.





      If we draw the graph, then the integral will be calculated should be



      $$ int_{0}^{2} int_{0}^{sqrt{4-x^{2}}} (4-y) : dy dx $$



      This would give the volume of the solid in 1st quadrant which can also be obtained through



      $$ int_{0}^{2} int_{0}^{sqrt{4-y^{2}}} (4-y) : dx dy $$



      which would be equal to $4pi - frac83$. Both the above equations give the same result.



      But if we try to find the volume of the entire solid formed by the three curves, then the results from the above equations(after changing the limits appropriately) don't match.
      $$ int_{-2}^{2} 2int_{0}^{sqrt{4-x^{2}}} (4-y) : dy dx = 2(8pi-frac{16}{3})$$
      $$ int_{-2}^{2} 2int_{0}^{sqrt{4-y^{2}}} (4-y) : dx dy = 16pi$$



      why the results of the these two equations don't match? Is there some problem with the limits I've set or something else? Please help



      PS: There is a similar question, but my query is different.







      multiple-integral






      share|cite|improve this question















      share|cite|improve this question













      share|cite|improve this question




      share|cite|improve this question








      edited Dec 23 '18 at 18:14







      Ajay Choudhary

















      asked Dec 23 '18 at 18:00









      Ajay ChoudharyAjay Choudhary

      888




      888






















          1 Answer
          1






          active

          oldest

          votes


















          1












          $begingroup$

          The limits of the second integral are wrong. It should be $$2int_0^2int_{-sqrt{4-y^{2}}}^{sqrt{4-y^{2}}} (4-y) : dx dy=4int_0^2int_0^{sqrt{4-y^{2}}} (4-y) : dx dy$$
          The answers match.






          share|cite|improve this answer











          $endgroup$













            Your Answer





            StackExchange.ifUsing("editor", function () {
            return StackExchange.using("mathjaxEditing", function () {
            StackExchange.MarkdownEditor.creationCallbacks.add(function (editor, postfix) {
            StackExchange.mathjaxEditing.prepareWmdForMathJax(editor, postfix, [["$", "$"], ["\\(","\\)"]]);
            });
            });
            }, "mathjax-editing");

            StackExchange.ready(function() {
            var channelOptions = {
            tags: "".split(" "),
            id: "69"
            };
            initTagRenderer("".split(" "), "".split(" "), channelOptions);

            StackExchange.using("externalEditor", function() {
            // Have to fire editor after snippets, if snippets enabled
            if (StackExchange.settings.snippets.snippetsEnabled) {
            StackExchange.using("snippets", function() {
            createEditor();
            });
            }
            else {
            createEditor();
            }
            });

            function createEditor() {
            StackExchange.prepareEditor({
            heartbeatType: 'answer',
            autoActivateHeartbeat: false,
            convertImagesToLinks: true,
            noModals: true,
            showLowRepImageUploadWarning: true,
            reputationToPostImages: 10,
            bindNavPrevention: true,
            postfix: "",
            imageUploader: {
            brandingHtml: "Powered by u003ca class="icon-imgur-white" href="https://imgur.com/"u003eu003c/au003e",
            contentPolicyHtml: "User contributions licensed under u003ca href="https://creativecommons.org/licenses/by-sa/3.0/"u003ecc by-sa 3.0 with attribution requiredu003c/au003e u003ca href="https://stackoverflow.com/legal/content-policy"u003e(content policy)u003c/au003e",
            allowUrls: true
            },
            noCode: true, onDemand: true,
            discardSelector: ".discard-answer"
            ,immediatelyShowMarkdownHelp:true
            });


            }
            });














            draft saved

            draft discarded


















            StackExchange.ready(
            function () {
            StackExchange.openid.initPostLogin('.new-post-login', 'https%3a%2f%2fmath.stackexchange.com%2fquestions%2f3050567%2ffind-the-volume-of-the-solid-bounded-by-the-xy-plane-the-cylinder-x2-y2%23new-answer', 'question_page');
            }
            );

            Post as a guest















            Required, but never shown

























            1 Answer
            1






            active

            oldest

            votes








            1 Answer
            1






            active

            oldest

            votes









            active

            oldest

            votes






            active

            oldest

            votes









            1












            $begingroup$

            The limits of the second integral are wrong. It should be $$2int_0^2int_{-sqrt{4-y^{2}}}^{sqrt{4-y^{2}}} (4-y) : dx dy=4int_0^2int_0^{sqrt{4-y^{2}}} (4-y) : dx dy$$
            The answers match.






            share|cite|improve this answer











            $endgroup$


















              1












              $begingroup$

              The limits of the second integral are wrong. It should be $$2int_0^2int_{-sqrt{4-y^{2}}}^{sqrt{4-y^{2}}} (4-y) : dx dy=4int_0^2int_0^{sqrt{4-y^{2}}} (4-y) : dx dy$$
              The answers match.






              share|cite|improve this answer











              $endgroup$
















                1












                1








                1





                $begingroup$

                The limits of the second integral are wrong. It should be $$2int_0^2int_{-sqrt{4-y^{2}}}^{sqrt{4-y^{2}}} (4-y) : dx dy=4int_0^2int_0^{sqrt{4-y^{2}}} (4-y) : dx dy$$
                The answers match.






                share|cite|improve this answer











                $endgroup$



                The limits of the second integral are wrong. It should be $$2int_0^2int_{-sqrt{4-y^{2}}}^{sqrt{4-y^{2}}} (4-y) : dx dy=4int_0^2int_0^{sqrt{4-y^{2}}} (4-y) : dx dy$$
                The answers match.







                share|cite|improve this answer














                share|cite|improve this answer



                share|cite|improve this answer








                edited Dec 23 '18 at 18:19

























                answered Dec 23 '18 at 18:06









                Shubham JohriShubham Johri

                5,204718




                5,204718






























                    draft saved

                    draft discarded




















































                    Thanks for contributing an answer to Mathematics Stack Exchange!


                    • Please be sure to answer the question. Provide details and share your research!

                    But avoid



                    • Asking for help, clarification, or responding to other answers.

                    • Making statements based on opinion; back them up with references or personal experience.


                    Use MathJax to format equations. MathJax reference.


                    To learn more, see our tips on writing great answers.




                    draft saved


                    draft discarded














                    StackExchange.ready(
                    function () {
                    StackExchange.openid.initPostLogin('.new-post-login', 'https%3a%2f%2fmath.stackexchange.com%2fquestions%2f3050567%2ffind-the-volume-of-the-solid-bounded-by-the-xy-plane-the-cylinder-x2-y2%23new-answer', 'question_page');
                    }
                    );

                    Post as a guest















                    Required, but never shown





















































                    Required, but never shown














                    Required, but never shown












                    Required, but never shown







                    Required, but never shown

































                    Required, but never shown














                    Required, but never shown












                    Required, but never shown







                    Required, but never shown







                    Popular posts from this blog

                    Wiesbaden

                    To store a contact into the json file from server.js file using a class in NodeJS

                    Marschland